LSAT and Law School Admissions Forum

Get expert LSAT preparation and law school admissions advice from PowerScore Test Preparation.

 Administrator
PowerScore Staff
  • PowerScore Staff
  • Posts: 8917
  • Joined: Feb 02, 2011
|
#40321
Complete Question Explanation
(The complete setup for this game can be found here: lsat/viewtopic.php?t=6748)

The correct answer choice is (E)

If T is the last report in the first segment, then S must be the last report in the second segment in order to comply with the fifth rule of the game:
PT72_Game_#1_#2_diagram 1.png
The setup shows that answer choice (E) is the correct answer choice to this Must Be True question.
You do not have the required permissions to view the files attached to this post.

Get the most out of your LSAT Prep Plus subscription.

Analyze and track your performance with our Testing and Analytics Package.